🧠 STUDY

GMAT 错题集

date
Apr 16, 2023
slug
wrong
author
status
Public
tags
📦 Set
🕌 Abroad
summary
题目来自 GMAT Official Guide 2022
type
Post
thumbnail
scream.png
category
🧠 STUDY
updatedAt
Jul 17, 2023 04:32 AM

Critical Reasoning

655

Loss of the Gocha mangrove forests has caused coastal erosion, reducing fish populations and requiring the Gocha Fishing Cooperative (GFC) to partially fund dredging and new shore facilities. However, as part of its subsidiary businesses, the GFC has now invested in a program to replant significant parts of the coast with mangrove trees. Given income from a controlled harvest of wood with continuing replanting, the mangrove regeneration effort makes it more likely that the cooperative will increase its net income.
Which of the following, if true, would most strengthen the argument that mangrove replanting will increase the Gocha cooperative’s net income?
A. The cost of dredging and shore facilities was shared with the local government.
B. The GFC will be able to hire local workers to assist with the mangrove replanting.
C. The GFC derives 10 percent of its revenue from salt-production facilities in an area previously cleared of mangroves.
D. Mangrove forests tend to increase the commercial fish populations in coastal fishing grounds.
E. A controlled harvesting of mangrove wood by the GFC would have little effect on coastal erosion.

661

Researchers asked volunteers to imagine they were running a five-kilometer race against 50 people and then against 500 people, races in each of which the top 10 percent would receive a $1,000 prize. Asked about the effort they would apply in the respective cases, the volunteers indicated , on average, that they would run slower in the race against the greater number of people. A likely explanation of this result is that those of the volunteers who were most comparatively inclined —those who most tended to compare themselves with others in the social environment—determined (perhaps unconsciously) that extreme effort would not be worthwhile in the 500-competitor race.
Which of the following would, if known to be true, most help justify the explanation offered above?
A. The volunteers who were most comparatively inclined were also those that had the greatest desire to win a $1,000 prize.
B. The volunteers who were the least comparatively inclined had no greater desire to win the $1,000 than those who were the most comparatively inclined .
C. The volunteers who were most comparatively inclined were likely to indicate that they would run the two races at the same speed.
D. The most comparatively inclined volunteers believed that they were significantly less likely to finish in the top 10 percent in the race against 500 than in the race against 50.
E. Volunteers were chosen for participation in the study on the basis of answers to various questions designed to measure the degree to which the volunteers were comparatively inclined .

691

Many agriculturally intensive areas of the world are beginning to encounter water scarcity problems. As a result, many farmers in these areas are likely to reduce their output as the water supply they need in order to maintain production shrinks . However, one group of farmers in such a region plans to increase their production by implementing techniques for water conservation.
Which of the following, it true, would most strongly support the prediction that the group's plan will succeed?
A. Farmers that can gain a larger share of the food market in their regions will be better positioned to control more water resources.
B. Most agricultural practices in areas with water shortages are water intensive .
C. Other regions of the world not facing water shortages are likely to make up for the reduction in agricultural output.
D. Demand for agricultural products in the group's region is not expected to decline.
E. More than half the water used for agriculture in the farmers’ region is lost to evaporation or leakage from irrigation channels.

693

A major network news organization experienced a drop in viewership in the week following the airing of a controversial report on the economy . The network also received a very large number of complaints regarding the report. The network, however, maintains that negative reactions to the report had nothing to do with its loss of viewers.
Which of the following, if true, most strongly supports the network's position?
A. The other major network news organizations reported similar reductions in viewership during the same week.
B. The viewers who registered complaints with the network were regular viewers of the news organization's programs.
C. Major network news organizations publicly attribute drops in viewership to their own reports only when they receive complaints about those reports.
D. This was not the first time that this network news organization has aired a controversial report on the economy that has inspired viewers to complain to the network.
E. Most network news viewers rely on network news broadcasts as their primary source of information regarding the economy .